Quinn CANNOT work in the same sales zone as which one of the following?

kens on June 12, 2020

June 2012 LSAT lg 17

Can someone explain this in more detail? Thanks!

Reply
Create a free account to read and take part in forum discussions.

Already have an account? log in

Skylar on June 12, 2020

@kenken, happy to help!

We can use the arrangements we came up with for the second question of this game to solve this question quickly.

Remember, the possible arrangements we came up with were:
1: PQSU
2: T
3: KM

and

1: PQKM
2: T
3: SU

This question asks who Quinn cannot work in the same zone with. In our first possible arrangement above, Q works with P, S, and U. This eliminates answer choices (C) and (E). In our second arraignment, Q works with P, K, and M. This eliminates answer choices (A) and (B). This leaves (D) as the correct answer choice.

(D) states that T cannot be in the same zone as Q. Looking at our rules, this makes sense because Rule #3 says that P and Q work together, and Rule #1 says that either P or T but not both must work in Zone 1.

Does that help? Please let us know if you have any other questions!